Đến nội dung

nhungvienkimcuong nội dung

Có 204 mục bởi nhungvienkimcuong (Tìm giới hạn từ 08-05-2020)



Sắp theo                Sắp xếp  

#741611 $ab^2c^3 \ge 1$

Đã gửi bởi nhungvienkimcuong on 04-10-2023 - 19:01 trong Bất đẳng thức - Cực trị

Cho $a,b,c>0$ thoả $a+b+c=\frac{1}{a} + \frac{1}{b} + \frac{1}{c}$. Chứng minh rằng nếu $a \le b \le c$ thì $ab^2c^3 \ge 1$

Để bỏ đi điều kiện đẳng thức của giả thiết ta đưa về bất đẳng thức thuần nhất, nghĩa là chứng minh

\[ab^2c^3\ge \left(\frac{abc(a+b+c)}{ab+bc+ca} \right )^3\iff (ab+bc+ca)^3\ge a^2b(a+b+c)^3.\]

Theo bất đẳng thức Cô-si thì $a^2b\le \left(\frac{2a+b}{3}\right)^3$, do đó ta chỉ cần chứng minh $ab+bc+ca\ge \frac{2a+b}{3}\cdot(a+b+c)$. Bất đẳng thức cuối luôn đúng vì

\begin{align*}3(ab+bc+ca)-(2a+b)(a+b+c)&=c(2b+a)-b^2-2a^2\\ &\ge b(2b+a)-b^2-2a^2=(b-a)(b+2a)\ge 0.\end{align*}




#741564 $ S=\begin{Bmatrix} 1,2,3,...,2n \end{Bmatrix...

Đã gửi bởi nhungvienkimcuong on 29-09-2023 - 21:56 trong Mệnh đề - tập hợp

Với mỗi tập hợp $M$, kí hiệu $\mathcal{P}(M)$ là họ tất cả tập con của $M$ (như vậy $|\mathcal{P}(M)|=2^{|M|}$).

 

Với mỗi tập con $A$ của $X$, ta phân hoạch $A=C\cup L$, trong đó $C$ gồm các phần tử chẳn và $L$ gồm các phần tử lẻ.

Với tập hợp $C$, ta thiết lập ánh xạ $\mathcal{P}\Big(\{2,4,\dots,2n-2,2n\}\Big)\to \mathcal{P}\Big(\{1,2,\dots,n\}\Big)$ như sau

\[C=\{c_1,c_2,\dots,c_k\}\longmapsto \underbrace{\{2,4,\dots,2n-2,2n\}\setminus\{c_1,c_2,\dots,c_k\}}_{\{d_1,d_2,\dots,d_{n-k}\}}\longmapsto \left \{ \frac{d_1}{2},\frac{d_2}{2},\dots,\frac{d_{n-k}}{2} \right \}.\]

Còn đối với tập $L$, ta thiết lập ánh xạ $\mathcal{P}\Big(\{1,3,\dots,2n-3,2n-1\}\Big)\to \mathcal{P}\Big(\{n+1,n+2,\dots,2n\}\Big)$ như sau

\[L=\{l_1,l_2,\dots,l_k\}\longmapsto \left \{ \frac{l_1+1}{2}+n,\frac{l_2+1}{2}+n,\dots,\frac{l_{k}+1}{2}+n \right \}.\]

Như vậy ta đã xây dựng một ánh xạ $X\to Y$ với

\[A=\{c_1,c_2,\dots,c_k,l_1,l_2,\dots,l_k\}\longmapsto \left \{ \frac{d_1}{2},\frac{d_2}{2},\dots,\frac{d_{n-k}}{2}, \frac{l_1+1}{2}+n,\frac{l_2+1}{2}+n,\dots,\frac{l_{k}+1}{2}+n \right \}.\]

Để chứng minh đây là một song ánh thì chỉ cần chứng tỏ mỗi bước thiết lập là song ánh, hoặc chứng tỏ $|X|=|Y|$ tương đương với

\[\sum_{k=0}^n\binom{n}{k}^2=\binom{2n}{n}.\]

 

 

Ghi chú. Một số bài xây dựng song ánh có thể kể đến GGTH 2017, Ấn Độ 2013 hoặc ở đây.




#741555 Tính tổng $\sum_{0\le k\le n/2} \frac...

Đã gửi bởi nhungvienkimcuong on 28-09-2023 - 16:42 trong Tổ hợp và rời rạc

Với $n$ là số nguyên dương, hãy tính tổng sau:
\[S_n=\sum_{k=0}^{\left\lfloor\frac{n}{2}\right\rfloor}\dfrac{(-1)^kn}{n-k}{n-k\choose k}\]

Xử lí tương tự ở đây, ta có

\[\sum_{n\ge 0}S_nx^n=\frac{1-x^2}{1-x+x^2}.\]

Nhờ thầy Thanh đã chỉ ra kết quả nên phần còn lại rất đơn giản. Chú ý rằng $\cos\frac{n\pi}{3}-\cos\frac{(n-1)\pi}{3}+\cos\frac{(n-2)\pi}{3}=0$, nên

\[2\sum_{n\ge 0}\cos\frac{n\pi}{3}x^n=\frac{2-x}{1-x+x^2}=1+\frac{1-x^2}{1-x+x^2}=1+S_0+\sum_{n\ge 1}S_nx^n.\]




#741554 Tính tổng $S_n=\sum_{k=0}^{\left\lfloor...

Đã gửi bởi nhungvienkimcuong on 28-09-2023 - 16:10 trong Tổ hợp và rời rạc

Bài toán
Với $n$ nguyên dương, tính tổng sau:
$S_n=\sum_{k=0}^{\left\lfloor\frac{n}{2}\right\rfloor} \frac{n}{n-k} {n-k\choose k}$

Trước tiên nhắc lại hai kết quả liên quan đến hàm sinh như sau:

  • Với $k$ là số tự nhiên thì $\frac{1}{(1-x)^{k+1}}=\sum_{n\ge 0}\binom{n+k}{k}x^n$,
  • $\sum_{n\ge 0}\sum_{k\ge 0}F(k,n)x^n=\sum_{k\ge 0}\sum_{n\ge 0}F(k,n)x^n$.

Giờ bắt đầu xử lí bài toán, xét hàm sinh $A(x)=\sum_{n\ge 0}S_nx^n$, ta có

$$\begin{align*}A(x)=\sum_{n\ge 0}\sum_{k\ge 0}\left(\binom{n-k}{k}+\binom{n-k-1}{k-1} \right )x^n=\underbrace{\sum_{n\ge 0}\sum_{k\ge 0}\binom{n-k}{k}x^n}_{B(x)}+\underbrace{\sum_{n\ge 0}\sum_{k\ge 0}\binom{n-k-2}{k}x^n}_{C(x)}.\end{align*}$$

Tiếp đến ta biến đổi $B(x)$ như sau

$$\begin{align*}B(x)&=\sum_{k\ge 0}\sum_{n\ge 0}\binom{n-k}{k}x^n= \sum_{k\ge 0}\sum_{n\ge 0}\binom{n+k}{k}x^{n+2k}=\sum_{k\ge 0}x^{2k}\sum_{n\ge 0}\binom{n+k}{k}x^n\\ &=\sum_{k\ge 0}x^{2k}\cdot \frac{1}{(1-x)^{k+1}}=\frac{1}{1-x}\sum_{k\ge 0}\left(\frac{x^2}{1-x} \right )^k=\frac{1}{1-x}\cdot\frac{1}{1-\frac{x^2}{1-x}}=\frac{1}{1-x-x^2}.\end{align*}$$

$C(x)$ thì hoàn toàn tương tự

$$\begin{align*}C(x)&=\sum_{k\ge 0}\sum_{n\ge 0}\binom{n-k-2}{k}x^n= \sum_{k\ge 0}\sum_{n\ge 0}\binom{n+k}{k}x^{n+2k+2}=\sum_{k\ge 0}x^{2k+2}\sum_{n\ge 0}\binom{n+k}{k}x^n\\ &=\sum_{k\ge 0}x^{2k+2}\cdot \frac{1}{(1-x)^{k+1}}=\frac{x^2}{1-x}\sum_{k\ge 0}\left(\frac{x^2}{1-x} \right )^k=\frac{x^2}{1-x}\cdot\frac{1}{1-\frac{x^2}{1-x}}=\frac{x^2}{1-x-x^2}.\end{align*}$$

Do vậy $A(x)=\frac{1+x^2}{1-x-x^2}$. Mặt khác ta biết rằng hàm sinh bởi dãy Lucas là $\sum_{n\ge 0}L_nx^n=\frac{2-x}{1-x-x^2}.$ Dẫn đến

\[A(x)=-1+\frac{2-x}{1-x-x^2}=-1+\sum_{n\ge 0}L_nx^n=1+\sum_{n\ge 1}L_nx^n.\]

 

 

Ghi chú. Tài liệu tham khảo về phương pháp này có Chuyên đề Đẳng thức tổ hợp của diễn đàn ta, hoặc Summation của Evan Chen.




#741546 Cho tập $X=\{1;2;3;\ldots ;3000\}$. Có tồn...

Đã gửi bởi nhungvienkimcuong on 27-09-2023 - 21:10 trong Tổ hợp và rời rạc

Phân hoạch tập hợp $X$ thành các tập con $X_{2i-1}$ như sau

$$X_{2i-1}=\Big\{2^j(2i-1)\mid j\in\mathbb{N},\ 2^j(2i-1)\le 3000\Big\},\quad i\in\{1,2,\dots,1500\}.$$

Từ đây ta thấy rằng để tập $A$ thỏa đề có số phần tử lớn nhất thì ở mỗi tập hợp $X_{2i-1}$ ta chọn $\left \lfloor \frac{|X_{2i-1}|+1}{2} \right \rfloor$ phần tử (không đồng thời chọn cả $x$ lẫn $2x$). Như vậy số phần tử lớn nhất của $A$ là

$$\sum_{i=1}^{1500}\left \lfloor \frac{|X_{2i-1}|+1}{2} \right \rfloor=\sum_{i=1}^{1500}\left \lfloor \frac{1}{2}\left \lfloor \log_2\left ( \frac{3000}{2i-1} \right )+1 \right \rfloor+\frac{1}{2} \right \rfloor.$$




#741545 $\frac{a}{b} + \frac{b}{c...

Đã gửi bởi nhungvienkimcuong on 27-09-2023 - 20:10 trong Bất đẳng thức - Cực trị

Cho $a,b,c>0 thoả \frac{1}{a+b+1} + \frac{1}{b+c+1} + \frac{1}{c+a+1}=1$. Chứng minh:
$\frac{a}{b} + \frac{b}{c} + \frac{c}{a} \ge a+b+c$

Ta có

\[1=\sum\frac{1}{a+b+1}=\sum\frac{a+b+c^2}{(a+b+1)(a+b+c^2)}\le \sum\frac{a+b+c^2}{(a+b+c)^2}=\frac{a^2+b^2+c^2+2(a+b+c)}{(a+b+c)^2}.\]

Suy ra $a+b+c\ge ab+bc+ca$, do vậy

\[\frac{a}{b}+\frac{b}{c}+\frac{c}{a}\ge \frac{(a+b+c)^2}{ab+bc+ca}\ge a+b+c.\]




#741536 $f:\mathbb{N^{*}}\rightarrow \mathbb...

Đã gửi bởi nhungvienkimcuong on 27-09-2023 - 10:16 trong Phương trình hàm

Tìm hàm $f:\mathbb{N^{*}}\rightarrow \mathbb{N^{*}}$ thỏa mãn $f(a)f(a+b)-ab$ là số chính phương với mọi a,b nguyên dương.

Bổ đề

Với mỗi số nguyên tố $p$ lẻ thì luôn tồn tại số nguyên $m$ sao cho $\left ( \frac{m}{p} \right )=-1$.

 

$\ast$ Chứng minh $f(1)\in \{1,2\}$.

 

$\ast$ Chứng minh $f(p)\mid 2p$ với mọi số nguyên tố $p$ lẻ.

 

$\ast$ Chứng minh $f(p)=p$ với mọi số nguyên tố $p\ge 7$.

Với $a:=1,b:=p-1$ thì $f(1)f(p)-p+1$ là số chính phương.

  • Nếu $f(p)\in \{1,2\}$ thì $f(1)f(p)-p+1\le 2\cdot 2-p+1=5-p$, từ đây dễ thấy mâu thuẫn.
  • Nếu $f(p)=2p$ thì $f(1)f(p)-p+1\in \{p+1,3p+1\}$. Tới đây giải phương trình nghiệm nguyên dễ thấy vô lí vì $p\ge 7$.

Vậy $f(p)=p$.

 

$\ast$ Chứng minh $f(n)=n$ với mọi số nguyên dương $n$.

Cố định $n$, xét số nguyên tố $p>\max\{7,f(n)+n\}$. Thay $a:=n,b:=p-n$ vào giả thiết thì

\[p(f(n)-n)+n^2=x^2.\]

Suy ra $p\mid x^2-n^2$ nên $x$ có dạng $kp\pm n$ với $k$ là số tự nhiên. Mặt khác với cách chọn $p$ thì

\[p(f(n)-n)+n^2<(p-n)^2\implies kp\pm n=x<p-n.\]

Do vậy $x=n$ nên $f(n)=n$.

 

 

Ghi chú. Những bài PTH về số chính phương thường khá khó, một số bài toán khác ở đâyđây.




#741503 Tìm đa thức hệ số nguyên $P(x)$ sao cho: $a^2 + ab +b^2|P(a)-P...

Đã gửi bởi nhungvienkimcuong on 24-09-2023 - 09:06 trong Đa thức

Bổ đề

Hai đa thức $G,H\in \mathbb{Z}[x]$ thỏa mãn $G$ là đa thức đơn khởi và $G(n)$ là ước của $H(n)$ với mọi số nguyên $n$. Khi đó đa thức $G$ là ước của đa thức $H$.

 

Dự đoán nghiệm là $P(x)=Q(x^3)$ nên ta viết lại đa thức $P$ dưới dạng

\[P(x)=Q_0(x^3)+xQ_1(x^3)+x^2Q_2(x^3)\]

với $Q_0,Q_1,Q_2\in \mathbb{Z}[x]$. Cố định số nguyên $a$, ta có

\begin{align*}P(x)&=Q_0(x^3)+xQ_1(x^3)+x^2Q_2(x^3)\\&=\underbrace{\sum_{i=0}^2x^i\big(Q_i(x^3)-Q_i(a^3)\big)}_{\vdots\  x^3-a^3}+Q_0(a^3)+xQ_1(a^3)+x^2Q_2(a^3)\end{align*}

Từ đây thấy rằng khi thực hiện phép chia $P(x)$ cho $x^2+xa+a^2$ thu được phần dư là

\[Q_0(a^3)+xQ_1(a^3)-(xa+a^2)Q_2(a^3).\]

Mặt khác kết hợp giả thiết với Theorem ta có $x^2+xa+a^2\mid P(x)-P(a)$ nên phần dư của phép chia $P(x)$ cho $x^2+xa+a^2$ phải là $P(a)$, do vậy $Q_0(a^3)+xQ_1(a^3)-(xa+a^2)Q_2(a^3)=P(a)$, tương đương

\[\Big(Q_1(a^3)-aQ_2(a^3)\Big)x-a^2Q_2(a^3)= 0\implies Q_1(a^3)-aQ_2(a^3)=a^2Q_2(a^3)=0.\]

Tới đây cho $a\to \infty$ suy ra $Q_1\equiv Q_2\equiv 0$.

 

 

Ghi chú. Hoàn toàn tương tự có thể giải quyết bài toán: Cho số nguyên dương $n$, tìm tất cả đa thức $P$ có hệ số nguyên thỏa mãn $a^n-b^n\mid P(a)-P(b)$ với mọi cặp số nguyên $a,b$.




#741499 $(k+1)x_{k+2}=\alpha x_{k+1}+(k-2022)x_{k...

Đã gửi bởi nhungvienkimcuong on 23-09-2023 - 21:36 trong Dãy số - Giới hạn

cho$\alpha$ là số thực và dãy số $(x_{n})$ được xác định như sau:$\left\{\begin{matrix} & x_{0}=0,x_{1}=1 \\  &(k+1)x_{k+2}=\alpha x_{k+1}+(k-2022)x_{k}\end{matrix}\right.$

Tìm giá trị $\alpha$ lớn nhất sao cho $x_{2023}=0$

Xét khai triển chuỗi lũy thừa $f(t)=x_1+x_2t+x_3t^2+x_4t^3+\dots$, từ giả thiết ta có

\begin{align*} \sum_{k=0}^{\infty}x_{k+2}\cdot(k+1)t^k&=\sum_{k=0}^{\infty}\alpha x_{k+1}t^k+\sum_{k=0}^{\infty}(k-2022)x_kt^k\\&=\alpha\sum_{k=0}^{\infty}x_{k+1}t^k+\sum_{k=1}^{\infty}(k-1)x_kt^k-2021\sum_{k=0}^{\infty}x_kt^k\\&=\alpha\sum_{k=0}^{\infty}x_{k+1}t^k+t^2\sum_{k=1}^{\infty}x_k\cdot(k-1)t^{k-2}-2021t\sum_{k=0}^{\infty}x_kt^{k-1}.\end{align*}
Do đó
\[f'(t)=\alpha f(t)+t^2f'(t)-2021tf(t)\implies \frac{f'(t)}{f(t)}=\frac{\alpha-2021t}{1-t^2}.\]
Từ đây nguyên hàm tìm được $f(t)=(1-t)^{\frac{2021-\alpha}{2}}(1+t)^{\frac{2021+\alpha}{2}}$. Để $f(t)$ là "đa thức" thì ta phải có $\frac{2021-\alpha}{2}$ và $\frac{2021+\alpha}{2}$ là các số tự nhiên. Với điều kiện này thấy ngay giá trị lớn nhất của $\alpha$ là $2021$ (chưa hề đụng tới $x_{2023}=0$).
Thử lại thỏa mãn, vì với $\alpha=2021$ thì $f(t)=(1+t)^{2021}=\sum_{k=0}^{2021}C_{2021}^kt^k+\sum_{k=2022}^{\infty}0\cdot x^k$, thấy ngay $x_{2023}=0$.
 
Ghi chú: Sử dụng hàm sinh thì diễn đàn ta có anh @supermember rất thượng thừa, có thể tham khảo ở đâyđây.



#741497 MIN $P=(a-b)(b-c)(c-a)(ab+bc+ca)$

Đã gửi bởi nhungvienkimcuong on 23-09-2023 - 12:05 trong Bất đẳng thức - Cực trị

Ta sẽ chứng minh $P\ge -4$. Dự đoán điểm rơi là $(a,b,c)\in \{(2,1,0),(0,-1,-2)\}$ để sử dụng Cô-si phù hợp.

Nếu $ab+bc+ca\le 0$ thì $P\ge 0>-4$, do đó chỉ cần xét khi $ab+bc+ca>0$.

\begin{equation*}(a-b)(b-c)\le \frac{(a-c)^2}{4}\Rightarrow -4P\le \underbrace{(a-c)^3(ab+bc+ca)}_{\mathcal{G}}.\end{equation*}

Tiếp tục thì Cô-si cho 5 số ta có

\begin{equation*}\mathcal{G}^2=8\left(\frac{(a-c)^2}{2}\right )^3(ab+bc+ca)^2\le 8\left(\frac{\frac{3}{2}(a-c)^2+2(ab+bc+ca)}{5} \right )^5.\end{equation*}

Cuối cùng thì chỉ cần chứng minh

\[8\left(\frac{\frac{3}{2}(a-c)^2+2(ab+bc+ca)}{5} \right )^5\le 16^2\iff 3(a-c)^2+4(ab+bc+ca)\le 20=4(a^2+b^2+c^2).\]

Bất đẳng thức cuối luôn đúng vì

\[3(a-c)^2+4(ab+bc+ca)\le 4(a^2+b^2+c^2)\iff (a-2b+c)^2\ge 0.\]

 

P/s. Một bài toán hơi tương tự tại đây.

Bài 55: Cho $x,y,z\geqslant 0$.Chứng minh rằng
$(x^2+y^2+z^2)^2\geqslant 4(x+y+z)(x-y)(y-z)(z-x)$




#740466 Đề thi Olympic Toán quốc tế (IMO) năm 2023

Đã gửi bởi nhungvienkimcuong on 08-07-2023 - 18:23 trong Thi HSG Quốc gia và Quốc tế

Đề thi Olympic Toán quốc tế năm 2023

Thời gian: 270 phút

 

Ngày thi thứ nhất: 08/07/2023

 

Bài 1. Xác định tất cả các hợp số $n>1$ thỏa mãn điều kiện sau: nếu $d_1,d_2, \dots, d_k$ là tất cả các ước nguyên dương của $n$ với $1=d_1<d_2<\dots<d_k=n$, thì $d_i$ là ước của $d_{i+1}+d_{i+2}$ với mọi $1\le i\le k-2$.

 

Bài 2. Cho tam giác nhọn $ABC$ với $AB<AC$. Gọi $\Omega$ là đường tròn ngoại tiếp của tam giác $ABC$. Gọi $S$ là điểm chính giữa cung $CB$ của $\Omega$ có chứa $A$. Đường thẳng vuông góc từ $A$ đến $BC$ cắt $BS$ tại $D$ và cắt lại $\Omega$ tại $E\neq A$. Đường thẳng qua $D$ song song với $BC$ cắt đường thẳng $BE$ tại $L$. Kí hiệu đường tròn ngoại tiếp của tam giác $BDL$ bởi $\omega$. Đường tròn $\omega$ cắt lại $\Omega$ tại $P\neq B$.

Chứng minh rằng đường tiếp tuyến của $\omega$ tại $P$ cắt đường thẳng $BS$ tại một điểm nằm trên đường phân giác trong của $\angle BAC$.

 

Bài 3. Với mỗi số nguyên $k\ge 2$, xác định tất cả các dãy vô hạn các số nguyên dương $a_1, a_2, \dots,$ để khi đó tồn tại một đa thức $P$ có dạng $P(x)=x^k+c_{k-1}x^{k-1}+\dots+c_1x+c_0$ với $c_0, c_1, \dots, c_{k-1}$ là các số nguyên không âm, sao cho

\[P(a_n)=a_{n+1}a_{n+2}\dots a_{n+k}\]

với mọi số nguyên $n\ge 1$.

 

 

Ngày thi thứ hai: 09/07/2023

 

Bài 4. Cho $x_1,x_2,\dots,x_{2023}$ là các số thực dương đôi một phân biệt sao cho

\[a_n=\sqrt{(x_1+x_2+\dots+x_n)\left(\frac{1}{x_1}+\frac{1}{x_2}+\dots+\frac{1}{x_n}\right)}\]

là một số nguyên với mọi $n=1,2,\dots,2023$. Chứng minh rằng $a_{2023}\ge 3034$.

 

Bài 5. Cho $n$ là một số nguyên dương. Một tam giác Nhật Bản gồm $1+2+\dots+n$ hình tròn được xếp thành một hình tam giác đều sao cho với mỗi $i=1,2,\dots,n$, hàng thứ $i$ có đúng $i$ hình tròn và trên hàng đó có đúng một hình tròn được tô màu đỏ. Một đường đi ninja trong một tam giác Nhật Bản là một dãy gồm $n$ hình tròn nhận được bằng cách xuất phát từ hàng trên cùng, đi lần lượt từ một hình tròn xuống một trong hai hình tròn ngay dưới nó, và kết thúc tại hàng dưới cùng. Trong hình vẽ là một tam giác Nhật Bản với $n=6$ và một đường đi ninja có chứa hai hình tròn màu đỏ.

Screenshot 2023-07-09 143751.png

Như một hàm số của $n$, tìm giá trị lớn nhất của $k$ sao cho trong mỗi tam giác Nhật Bản luôn có một đường đi ninja chứa ít nhất $k$ hình tròn màu đỏ.

 

Bài 6. Cho tam giác đều $ABC$. Các điểm $A_1,B_1,C_1$ nằm trong tam giác $ABC$ sao cho $BA_1=A_1C,CB_1=B_1A,AC_1=C_1B$ và

\[\angle BA_1C+\angle CB_1A+\angle AC_1B=480^{\circ}.\]

Cho $BC_1$ và $CB_1$ cắt nhau tại $A_2$, $CA_1$ và $AC_1$ cắt nhau tại $B_2$, $AB_1$ và $BA_1$ cắt nhau tại $C_2$.

Chứng minh rằng nếu $A_1B_1C_1$ là tam giác không cân thì các đường tròn ngoại tiếp của ba tam giác $AA_1A_2,BB_1B_2$ và $CC_1C_2$ sẽ đi qua hai điểm chung.

 

 

Nguồn: imo-official




#738575 Đề thi chọn đội tuyển Olympic quốc tế (TST) năm 2023

Đã gửi bởi nhungvienkimcuong on 13-04-2023 - 15:32 trong Thi HSG Quốc gia và Quốc tế

Đề thi chọn đội tuyển Olympic quốc tế năm 2023

Thời gian: 270 phút

 

Ngày thi thứ nhất: 13/04/2023

 

Bài 1. Cho hai lớp học, lớp $A$ có $m$ học sinh và lớp $B$ có $n$ học sinh $(m,\ n>1)$. Học sinh của hai lớp ngồi quanh một bàn tròn và mỗi em học sinh X được cô giáo phát kẹo bằng với số bạn ngồi liên tiếp kề bên trái X và cùng lớp với X (nếu X không có những bạn như vậy thì X không có kẹo). Những người có cùng số kẹo được cô giáo phân chia vào cùng một nhóm.

a) Hỏi số người đông nhất của một nhóm có thể là bao nhiêu?

b) Nếu không xét nhóm mà học sinh không có kẹo thì số người đông nhất của một nhóm có thể là bao nhiêu?

 

Bài 2. Xét các hàm số sau đây trên tập số thực khác $0$:

\[P(x)=\left(x^2-1\right)^{2023},\quad Q(x)=(2x+1)^{14},\quad R(x)=\left(2x+1+\frac{2}{x}\right)^{34}.\]

Giả sử ban đầu có một danh sách gồm đúng hai hàm trong các hàm đã cho. Mỗi thao tác được phép cộng, trừ, nhân các hàm trong danh sách đó lại với nhau (hoặc lấy lũy thừa với số mũ nguyên dương một hàm trong đó). Ta cũng có thể cộng, trừ, nhân một hàm với một số thực tùy ý để tạo ra hàm mới và đưa vào danh sách. Quá trình trên có thể thực hiện nhiều lần. Chứng minh rằng từ danh sách ban đầu là hai hàm bất kì trong ba hàm đã cho, ta không thể thu được hàm còn lại.

 

Bài 3. Cho tam giác $ABC$ nhọn không cân nội tiếp đường tròn $(O)$. Các đường cao $BE,\ CF$ của tam giác $ABC$ cắt nhau ở trực tâm $H$ và $M$ là trung điểm $AH$. Gọi $K$ là hình chiếu của $H$ lên $EF$. Đường thẳng không đi qua $A$ và song song với $BC$ cắt cung nhỏ $AB,\ AC$ lần lượt tại các điểm $P,\ Q$. Chứng minh rằng tiếp tuyến tại $E$ của đường tròn ngoại tiếp tam giác $CQE$ và tiếp tuyến tại $F$ của đường tròn ngoại tiếp tam giác $BPF$ cắt nhau trên đường thẳng $MK$.

 

 

Ngày thi thứ hai: 14/04/2023

 

Bài 4. Cho hai số nguyên dương $a,\ b$ nguyên tố cùng nhau với $b$ lẻ và $a>2$. Xét dãy số $(x_n)$ xác định bởi $x_0=2,\ x_1=a$ và $x_{n+2}=ax_{n+1}+bx_n$ với mọi $n$ nguyên dương. Chứng minh rằng

a) Nếu $a$ chẵn thì không tồn tại các số nguyên dương $m,\ n,\ p$ để $\frac{x_m}{x_nx_p}$ là số nguyên.

b) Nếu $a$ lẻ thì không tồn tại các số nguyên dương $m,\ n,\ p$ sao cho $mnp$ chẵn và $\frac{x_m}{x_nx_p}$ là số chính phương.

 

Bài 5. Cho tứ giác lồi $ABCD$ có $\widehat{B}<\widehat{A}<90^{\circ}$. Gọi $I$ là trung điểm của $AB$ và $S$ là giao điểm của $AD$ với $BC$. Xét $R$ là một điểm thay đổi nằm bên trong tam giác $SAB$ sao cho $\widehat{ASR}=\widehat{BSR}$. Trên các đường thẳng $AR,\ BR$ lần lượt lấy các điểm $E,\ F$ sao cho $BE$ và $AF$ cùng song song với $RS$. Giả sử $EF$ cắt đường tròn ngoại tiếp tam giác $SAB$ tại các điểm $H,\ K$. Trên đoạn $AB$, lấy các điểm $M$ và $N$ sao cho $\widehat{AHM}=\widehat{BHI}$ và $\widehat{BKN}=\widehat{AKI}$.

a) Chứng minh rằng tâm $J$ của đường tròn ngoại tiếp tam giác $SMN$ thuộc một đường thẳng cố định.

b) Trên $BE,\ AF$ lần lượt lấy các điểm $P,\ Q$ sao cho $CP$ song song với $SE$ và $DQ$ song song với $SF$. Các đường thẳng $SE,\ SF$ cắt lại đường tròn $(O)$ theo thứ tự tại $U,\ V$. Gọi $G$ là giao điểm của $AU$ với $BV$. Chứng minh rằng đường trung tuyến đỉnh $G$ của tam giác $GPQ$ luôn đi qua một điểm cố định.

 

Bài 6. Cho số nguyên $n\ge 3$ và tập hợp $A=\{1,2,\dots,n\}$. Xác định số $k$ lớn nhất sao cho với mỗi bộ $k$ tập con có $3$ phần tử của $A$, luôn tô màu được mỗi phần tử của $A$ bởi một trong hai màu xanh hoặc đỏ (mỗi phần tử một màu) để không có tập con nào trong $k$ tập con trên có ba phần tử cùng màu.

 

 

Nguồn: Hướng tới Olympic Toán VN (nhóm facebook)




#737712 [HOT HOT HOT] ĐÃ CÓ KẾT QUẢ THI HSG QUỐC GIA 2023

Đã gửi bởi nhungvienkimcuong on 14-03-2023 - 04:56 trong Thi HSG Quốc gia và Quốc tế

Bạn thủ khoa này không lạ mặt gì với diễn đàn cả, chúc mừng @Hoang72




#737424 Đề thi học sinh giỏi quốc gia (VMO) năm 2023

Đã gửi bởi nhungvienkimcuong on 24-02-2023 - 14:56 trong Thi HSG Quốc gia và Quốc tế

Đề thi học sinh giỏi quốc gia năm 2023

Thời gian: 180 phút

 

Ngày thi thứ nhất: 24/02/2023

 

Bài 1 (5 điểm)

Xét dãy số $(a_n)$ thỏa mãn $a_1=\frac{1}{2},\ a_{n+1}=\sqrt[3]{3a_{n+1}-a_n}$ và $0\le a_n\le 1$, với mọi $n\ge 1$.

a) Chứng minh rằng dãy $(a_n)$ xác định duy nhất và có giới hạn hữu hạn.

b) Cho dãy số $(b_n)$ xác định bởi $b_n=(1+2a_1)(1+2^2a_2)\cdots(1+2^na_n)$ với mọi $n\ge 1$. Chứng minh rằng dãy $(b_n)$ có giới hạn hữu hạn.

 

Bài 2 (5 điểm)

Cho các số nguyên $a,\ b,\ c,\ \alpha,\ \beta$ và dãy số $(u_n)$ xác định bởi

\[u_1=\alpha,\ u_2=\beta,\ u_{n+2}=au_{n+1}+bu_n+c\ \text{ với mọi }\ n\ge 1.\]

a) Chứng minh rằng nếu $a=3,\ b=-2,\ c=-1$ thì có vô số cặp số nguyên $(\alpha;\beta)$ để $u_{2023}=2^{2022}$.

b) Chứng minh rằng tồn tại số nguyên dương $n_0$ sao cho có duy nhất một trong hai khẳng định sau là đúng:

    i)  Có vô số số nguyên dương $m$ để $u_{n_0}u_{n_0+1}\cdots u_{n_0+m}$ chia hết cho $7^{2023}$ hoặc $17^{2023}$;

    ii) Có vô số số nguyên dương $k$ để $u_{n_0}u_{n_0+1}\cdots u_{n_0+k}-1$ chia hết cho $2023$.

 

Bài 3 (5 điểm)

Tìm số thực dương $k$ lớn nhất sao cho bất đẳng thức

\[\frac{1}{kab+c^2}+\frac{1}{kbc+a^2}+\frac{1}{kca+b^2}\ge \frac{k+3}{a^2+b^2+c^2}\]

đúng với mọi bộ ba số thực dương $(a;\ b;\ c)$ thỏa mãn điều kiện $a^2+b^2+c^2=2(ab+bc+ca)$.

 

Bài 4 (5 điểm)

Cho tứ giác $ABCD$ có $DB=DC$ và nội tiếp một đường tròn. Gọi $M,\ N$ tương ứng là trung điểm của $AB,\ AC$ và $J,\ E,\ F$ tương ứng là các tiếp điểm của đường tròn $(I)$ nội tiếp tam giác $ABC$ với $BC,\ CA,\ AB$. Đường thẳng $MN$ cắt $JE,\ JF$ lần lượt tại $K,\ H$; $IJ$ cắt lại đường tròn $(IBC)$ tại $G$ và $DG$ cắt lại $(IBC)$ tại $T$.

a) Chứng minh rằng $JA$ đi qua trung điểm của $HK$ và vuông góc với $IT$.

b) Gọi $R,\ S$ tương ứng là hình chiếu vuông góc của $D$ trên $AB,\ AC$. Lấy các điểm $P,\ Q$ lần lượt trên $IF,\ IE$ sao cho $KP$ và $HQ$ đều vuông góc với $MN$. Chứng minh rằng ba đường thẳng $MP,\ NQ$ và $RS$ đồng quy.

 

 

Ngày thi thứ hai: 25/02/2023

 

Bài 5 (6 điểm)

Xét các hàm số $f\colon \mathbb{R}\to\mathbb{R}$ và $g\colon \mathbb{R}\to\mathbb{R}$ thỏa mãn điều kiện $f(0)=2022$ và

\[f(x+g(y))=xf(y)+(2023-y)f(x)+g(x)\ \text{ với mọi }\ x,y\in\mathbb{R}.\]

a) Chứng minh rằng $f$ là một toàn ánh và $g$ là một đơn ánh.

b) Tìm tất cả các hàm số $f$ và $g$ thỏa mãn điều kiện bài toán.

 

Bài 6 (7 điểm)

Có $n\ge 2$ lớp học tổ chức $m\ge 1$ tổ ngoại khóa cho học sinh. Lớp nào cũng có học sinh tham gia ít nhất một tổ ngoại khóa. Mọi tổ ngoại khóa đều có đúng $a$ lớp có học sinh tham gia. Với hai tổ ngoại khóa bất kỳ, có không quá $b$ lớp có học sinh tham gia đồng thời cả hai tổ này.

a) Tính $m$ khi $n=8,\ a=4,\ b=1$.

b) Chứng minh rằng $n\ge 20$ khi $m=6,\ a=10,\ b=4$.

c) Tìm giá trị nhỏ nhất của $n$ khi $m=20,\ a=4,\ b=1$.

 

Bài 7 (7 điểm)

Cho tam giác nhọn, không cân $ABC$ có trực tâm $H$ và tâm đường tròn ngoại tiếp $O$. Đường tròn nội tiếp $(I)$ của tam giác $ABC$ tiếp xúc với các cạnh $BC,\ CA,\ AB$ tương ứng tại $M,\ N,\ P$. Gọi $\Omega_A$ là một đường tròn đi qua $A$, tiếp xúc ngoài với $(I)$ tại một điểm $A'$ và cắt lại $AB,\ AC$ tương ứng tại $A_b,\ A_c$. Các đường tròn $\Omega_B,\ \Omega_C$ và các điểm $B',\ B_a,\ B_c,\ C',\ C_a,\ C_b$ được xác định một cách tương tự.

a) Chứng minh rằng $B_cC_b+C_aA_c+A_bB_a\ge NP+PM+MN$.

b) Xét trường hợp $A',\ B',\ C'$ tương ứng thuộc các đường thẳng $AM,\ BN,\ CP$. Gọi $K$ là tâm đường tròn ngoại tiếp tam giác có ba cạnh tương ứng thuộc ba đường thẳng $A_bA_c,\ B_cB_a,\ C_aC_b$. Chứng minh rằng $OH$ song song với $IK$.

 

 

Nguồn: VnExpress (ngày 1, ngày 2)




#736701 Đề thi Vòng 1 Liên hiệp Anh (BMO 1) năm 2023

Đã gửi bởi nhungvienkimcuong on 10-01-2023 - 09:07 trong Thi HSG Quốc gia và Quốc tế

Vòng 1 Liên hiệp Anh năm 2023

Thời gian: 3 tiếng 30 phút


Bài 1. Một con đường có các ngôi nhà được đánh số từ $1$ tới $n$, với $n$ là một số có ba chữ số. Biết rằng có đúng $1/k$ các số bắt đầu bằng chữ số $2$, với $k$ là một số nguyên dương. Xác định các giá trị có thể của $n$.

Bài 2. Một dãy số nguyên dương $(a_n)$ thỏa mãn: với mọi số nguyên dương $n$ thì

$a_{2n+1}=a_{2n}a_{2n-1},\quad a_{2n+2}=a_{2n+1}+4.$

Có đúng $m$ số trong các số $a_1,\ a_2,\ \dots,\ a_{2022}$ là số chính phương. Giá trị lớn nhất của $m$ là bao nhiêu?

Bài 3. Cho tam giác nhọn $ABC$ không cân, trung điểm của $AC$ và $AB$ lần lượt là $B_1$ và $C_1$. Một điểm $D$ nằm trên $BC$ sao cho $C$ nằm giữa $B$ và $D$. Điểm $F$ thỏa mãn $\angle AFC$ vuông và $\angle DCF=\angle FCA$. Điểm $G$ thỏa mãn $\angle AGB$ vuông và $\angle CBG=\angle GBA$. Chứng minh rằng $B_1,\ C_1,\ F$ và $G$ thẳng hàng.

Bài 4. Alex và Katy chơi một trò chơi trên bảng $8\times 8$ có $64$ ô vuông đơn vị. Họ chơi theo lượt với Alex là người chơi trước. Ở lượt của Alex, cậu ấy viết 'A' vào một ô trống. Ở lượt của Katy, cô ấy viết 'K' vào hai ô trống có cùng cạnh. Trò chơi kết thúc khi có một người không thể thực hiện lượt đi. Điểm của Katy là số kí tự K trong bảng khi trò chơi kết thúc. Bất kể Alex làm gì thì điểm số cao nhất mà Katy chắc chắn nhận được là bao nhiêu?

Bài 5. Với mỗi số nguyên $n\ge 1$, kí hiệu $f(n)$ là số lượng các số có các chữ số khác nhau bắt đầu bởi $1$ và kết thúc bởi $n$, ngoài ra mỗi chữ số là ước của chữ số liền sau nó. Chứng minh rằng với mỗi số nguyên $N\ge 1$ thì luôn tồn tại số nguyên $n$ sao cho $N$ là ước của $f(n)$.
($f(1)=1,\ f(2)=1$ và $f(6)=3$)

Bài 6. Cho đường tròn $\Gamma$ có bán kính $1$. Một đường thẳng $l$ cách tâm của $\Gamma$ một khoảng nằm giữa $0$ và $2$. Một con ếch ngồi trên một điểm thuộc $\Gamma$ sao cho khoảng cách so với $l$ bé hơn $1$. Sau đó con ếch thực hiện liên tiếp các cú nhảy. Mỗi cú nhảy có độ dài $1$, nếu bắt đầu trên $\Gamma$ thì phải kết thúc trên $l$ và ngược lại. Chứng minh rằng sau một số lần nhảy thì con ếch sẽ quay lại điểm bắt đầu.

Nguồn: https://bmos.ukmt.or.../home/bmo.shtml




#736696 Cho đa giác đều $2018$ đỉnh,chọn ngẫu nhiên 3 đỉnh.Tính xác suất ch...

Đã gửi bởi nhungvienkimcuong on 09-01-2023 - 18:29 trong Tổ hợp - Xác suất và thống kê - Số phức

Câu 15 trong file sau trình bày hai cách khác File gửi kèm  de KSCL Binh Phu.pdf   816.9K   113 Số lần tải




#736645 CM $\exists g:\mathbb R\mapsto \mathbb R$ thỏa...

Đã gửi bởi nhungvienkimcuong on 06-01-2023 - 08:44 trong Phương trình - Hệ phương trình - Bất phương trình

Với $\varepsilon >0$ cho trước. Hàm $f:\mathbb R\mapsto \mathbb R$ thỏa mãn

$$\left|f(x+y)-f(x-y)-2f(y)\right|\le \varepsilon,\forall x,y\in\mathbb R.$$
Chứng minh rằng: $\exists$ hàm $g:\mathbb R\mapsto \mathbb R$ cộng tính sao cho $\left|f(x)-g(x)\right|\le \varepsilon$

$\bullet$ Cố định $t$, chứng minh tồn tại $\lim_{n\to \infty}\frac{f(3^nt)}{3^n}$.

Xây dựng hàm $g$ như sau

\[\begin{array}{rccl}g\colon &\mathbb{R}&\to &\mathbb{R}\\& t & \mapsto& \displaystyle\lim_{n\to \infty}\frac{f(3^nt)}{3^n}\end{array}\]

$\bullet$ Chứng minh $g$ cộng tính.

 

$\bullet$ Chứng minh $|f(x)-g(x)|\le \varepsilon$.




#736644 $\sum_{i=1}^n(a_i^7+a_i^3)\ge 2\left(\sum_...

Đã gửi bởi nhungvienkimcuong on 06-01-2023 - 06:54 trong Bất đẳng thức - Cực trị

Ngớ ngẩn thật, tiêu đề lẫn nội dung đều viết sai  :( . Đề đúng là như sau:

Cho $n$ số nguyên dương $a_1,\ a_2,\ \dots,\ a_n$ phân biệt. Chứng minh rằng
\[a_1^7+a_1^{\color{red}5}+a_2^7+a_2^{\color{red}5}+\dots+a_n^7+a_n^{\color{red}5}\ge 2\left(a_1^3+a_2^3+\dots+a_n^3\right)^2.\]




#736637 $P(a)+P(b)$ cũng là số chính phương.

Đã gửi bởi nhungvienkimcuong on 05-01-2023 - 15:07 trong Đa thức

Bài này từng rất mất thời gian trong VMF's Marathon Đa thức, xem ở đây.




#736635 Chứng minh rằng tồn tại các số nguyên dương $N$ sao cho với mọi số...

Đã gửi bởi nhungvienkimcuong on 05-01-2023 - 08:26 trong Phương trình hàm

Dễ dàng có $f(1)=1$. Với $p$ là số nguyên tố, thay $m:=p,\ n:=1$ thì
\[f(p)+1-p\mid pf(p)+1\implies f(p)+1-p\mid p^2-p+1.\tag{$\ast$}\]
Thay $m:=p,\ n:=p$ thì
\[2f(p)-p^2\mid 2pf(p)\implies 2f(p)-p^2\mid p^3.\]
Từ đây suy ra $2f(p)-p^2\in \{1,p,p^2,p^3\}$, xét từng trường hợp kết hợp với $(\ast)$ suy ra $f(p)=p^2$ với $p$ đủ lớn.

Ghi chú. Có thể chứng minh $f(n)=n^2$ với mọi số nguyên dương $n$.



#736634 Công sai $d$ chia hết cho mỗi số nguyên tố $q < n$

Đã gửi bởi nhungvienkimcuong on 05-01-2023 - 07:45 trong Dãy số - Giới hạn

Chứng minh rằng nếu tất cả $n > 2$ số hạng của dãy cấp số cộng $$p, p+d, p+2d,\ldots,p+(n-1)d$$ là số nguyên tố, thì công sai $d$ chia hết cho mỗi số nguyên tố $q < n$.

Nếu $n>p$ thì phần tử $p+p\cdot d$ trong dãy là hợp số (vô lí). Do vậy $n\le p$.

Giả sử tồn tại số nguyên tố $q<n$ sao cho $q\nmid d$. Khi đó $\text{UCLN}(q,d)=1$ nên sẽ tồn tại một số trong các số

\[p,p+d,\dots,p+(q-1)d\]

là bội của $q$, giả sử là $p+kd$. Mặt khác $q<n\le p\le p+kd$ nên $p+kd$ không thể là số nguyên tố (mâu thuẫn).




#736631 Chứng minh rằng với mọi cách lấy $n$ phần tử của $S$ bao...

Đã gửi bởi nhungvienkimcuong on 04-01-2023 - 21:26 trong Tổ hợp và rời rạc

Gọi $A$ là tập hợp chứa $n$ phần tử được chọn. Gọi $m$ là phần tử nhỏ nhất của $A$ và giả sử $m\le n$. Nếu $2m\in A$ thì cặp $(m,2m)$ thỏa đề. Nếu $2m\notin A$, xét tập hợp $A'=(A\setminus\{m\})\cup\{2m\}$. Với mỗi tập hợp $X$, kí hiệu

\[\omega(X)=\min\Big\{\text{BCNN}(a,b):a,b\in X\Big\}.\]

Khi đó $\omega(A)\le \omega(A')$, như vậy nếu mệnh đề đúng với $A'$ thì cũng đúng với $A$.

 

Vậy ta chỉ cần xử lí trong trường hợp $\omega(A)$ lớn nhất, đó là khi $A=\{n+1,n+2,\dots,2n\}$. Tới đây chỉ cần chia ra hai trường hợp chẵn lẻ của $n$.




#736265 $x+y+z\ge 3\sqrt[3]{2xyz}$

Đã gửi bởi nhungvienkimcuong on 15-12-2022 - 21:05 trong Bất đẳng thức và cực trị

Cho $x,y,z$ không âm thỏa mãn $x^2+y^2+z^2=2\left(xy+yz+zx\right)$

Chứng minh $x+y+z\ge 3\sqrt[3]{2xyz}$

Từ giả thiết có được $(y+z-x)^2=4yz$. Giả sử $x=\min(x,y,z)$ thì suy ra $y+z-x=2\sqrt{yz}$. Do đó

\[x+y+z=2(x+\sqrt{yz})=2\left ( x+\frac{\sqrt{yz}}{2}+\frac{\sqrt{yz}}{2} \right )\ge 6\sqrt[3]{\frac{xyz}{4}}.\]

 

P/s. Giờ THCS học khó thể nhỉ  :mellow:. Thấy các bạn đăng những bài khó ghê, nghĩ lại lúc tuổi mình bằng các bạn thì gặp như thế chẳng dám đụng cơ  :icon6:




#736264 Chứng minh rằng tồn tại số nguyên $a$, $1<a<\frac...

Đã gửi bởi nhungvienkimcuong on 15-12-2022 - 20:58 trong Số học

Bạn có thể chia sẻ quá trình nghĩ ra lời giải này không nhỉ? Cám ơn bạn nhé!

Điều kiện chia hết đơn giản, điều kiện còn lại khá khó nên mình cần xem xét tất cả số $a$ thỏa mãn $n\mid a^2-a$. Có đúng $2^k$ số (xét theo modulo $n$) thỏa mãn $n\mid a^2-a$, chính là các số có dạng $\sum \epsilon_ix_i$ với $\epsilon_i\in \{0,1\}$.

 

Điều kiện còn lại thì mình nghĩ tới việc chỉ ra hai số cùng thuộc $\left[ \frac{jn}{k},\frac{(j+1)n}{k} \right )$ nên xét $k+1$ số $X_i$ như trên. Phần còn lại là làm sao cho hợp lí, dẫn đến việc cần chứng minh nhận xét.




#736263 $\sum_{i=1}^kx_i^2<\frac{1}{2...

Đã gửi bởi nhungvienkimcuong on 15-12-2022 - 20:46 trong Số học

Bài này gớm thật, đáp số là $516$ (xem ở đây)